SECTION 1

Time-35 minutes

23 Questions

Directions: Each group of questions in this section is based on a set of conditions. In answering some of the questions, it may be useful to draw a rough diagram. Choosethe response that most accurately and completely answers each question and blacken the corresponding space on your answer sheet.

Question 1-6

A man buys three outfits-X, Y, and Z-each of which consists of two articles of clothing.

Each of the articles of clothing is either brown gray or navy.

At least one of the outfits is made up of two articles different in color from one another.

No more than two of the outfits contain the same combination of colors.

Outfit X contains at least one navy article of clothing.

Outfit Y contains at least one brown article of clothing and does not contain a gray article.

1. Which one of the following can be the colors of the man's outfits?

(A) X: gray and navy; Y: brown and gray; Z: gray and gray

(B) X: brown and gray; Y: brown and navy; Z: gray and gray

(C) X: navy and navy; Y: brown and brown; Z: brown and navy

(D) X: brown and navy; Y: brown and navy; Z: brown and navy

(E) X: navy and navy; Y: brown and brown; Z: navy and navy

2. If outfits X and Y each consist of one brown article and one navy article of clothing, what combinations for outfit Z?

(A) 2

(B) 3

(C) 4

(D) 5

(E) 6

3. If outfit Z does not contain two brown items of clothing, what is the maximum number of items of clothing in the three outfits that can be navy?

(A) 1

(B) 2

(C) 3

(D) 4

(E) 5

4. If outfit Y consists of two brown articles of clothing and outfit Z consists of two navy items, what is the total number of possible color combinations for outfit X?

(A) 1

(B) 2

(C) 3

(D) 4

(E) 5

5. Which one of the following color combinations for outfit Z would be acceptable under any of the acceptable color combinations for outfits X and Y?

(A) gray and gray

(B) brown and gray

(C) brown and brown

(D) brown and navy

(E) navy and navy

6. If no two outfits contain the same color combination but each contains at least one navy item, which one of the following is a complete and accurate list of the possiblecombinations for outfit X?

(A) gray and navy

(B) brown and navy

(C) navy and navy

(D) gray and navy; brown and navy

(E) gray and navy; navy and navy

Question 7-11

Five seats on a train are arranged in two rows that face each other. The seats in row 1 are seat 1 and seat 2. The seats in row 2 are seat 3, seat 4, and seat 5 in that order. Seat 1 is directly across from seat 3. Seat 2 is directly across from seat 4. Seat 5 is not directly across from any other seat. Three women-Allice, Betty, and Carol-and twomen-David, Edmund-each must be seated in one of the five seats, one person to a seat.

Betty sits in seat 4.

Carol sits next to neither Betty nor David.

Alice does not sit directly across from Carol.

7. If Edmund sits directly across from Betty, which one of the following must be true?

(A) Alice does not sit directly across from anyone.

(B) Alice sits directly across from David.

(C) Alice sits next to Edmund.

(D) David sits next to Alice.

(E) David sits next to Edmund.

8. If Edmund does not sit directly across from anyone, which one of the following is a complete and accurate list of those (other than Edmund) who cannot sit in seat 1?

(A) Betty

(B) Carol

(C) David

(D) Betty and Carol

(E) Betty and David

9. If none of the five passengers sits directly across from a passenger of the same sex, how many seating arrangements of the five passengers are possible?

(A) exactly 0

(B) exactly 1

(C) exactly 2

(D) exactly 3

(E) exactly 4

10. How many of the seats could be the one Alice selects as her seat?

(A) 1 only

(B) 2 only

(C) 3 only

(D) 4 only

(E) 5 only

11. How many different seating arrangements of the passengers might there be for row 2?

(A) 2

(B) 3

(C) 4

(D) 5

(E) 6

Question 12-17

Six people-Julio, Kevin, May, Norma, Olivia, and Tamio-participate in a track meet. Two of them enter the marathon, two enter the relay, and two enter the sprint.

Each participant enters only one event.

If Kevin enters the marathon, then both Julio and

May enter the relay, and if both Julio and May enter the relay, then Kevin enters the marathon.

If Norma does not enter the sprint, then Tamio enters the relay.

If Olivia enters the relay, then Julio does not enter the relay.

Olivia and Tamio do not both enter the relay.

12. If Kevin enters the marathon, then which one of the following cannot be true?

(A) Julio enters the relay and Norma enters the sprint.

(B) Olivia enters the marathon and Norma enters the sprint.

(C) Tamio enters the marathon and Olivia enters the sprint.

(D) Tamio enters the relay and Olivia enters the sprint.

(E) Tamio enters the marathon and May enters the sprint.

13. If Norma enters the marathon, then which one of the following cannot be true?

(A) Both Kevin and Olivia enter the sprint.

(B) Kevin enters the relay and Olivia enters the sprint.

(C) Kevin enters the marathon and May enters the relay.

(D) Julio enters the marathon and May enters the relay.

(E) Julio enters the marathon and Kevin enters the relay.

14. If both Kevin and Olivia enter the relay, then which one of the following must be true?

(A) Julion and Tamio enter the marathon.

(B) May enters the marathon.

(C) May enters the sprint.

(D) Tamio enters the sprint.

(E) Norma enters the sprint.

15. If both Julio and Kevin enter the same event, then which of the following can be true?

Ⅰ.Julio enters the relay.

Ⅱ. May enters the marathon.

Ⅲ. Olivia enters the relay.

(A) Ⅰonly

(B) Ⅱonly

(C) Ⅲonly

(D) Ⅰand Ⅱonly

(E) Ⅰ,Ⅱ,and Ⅲ

16. If Norma and Olivia each enter different events, then which one of the following cannot be true?

(A) Kevin and Olivia enter the marathon.

(B) Julio and Norma enter the relay.

(C) May and Olivia enter the relay.

(D) Norma and Tamio enter the relay.

(E) Olivia enters the sprint and Tamio enters the relay.

17. If both Julio and Kevin enter the relay, then which one of the following must be false?

(A) May and Norma enter the marathon.

(B) May and Olivia enter the marathon.

(C) Norma and Tamio enter the sprint.

(D) May and Norma enter the sprint.

(E) May enters the sprint and Tamio enters marathon.

Questions 18-23

Seven lights-J, K, L, M, N, O, and P-have only two possible settings: on or off. They are arranged on a particular electric circuit as follows.

If K is on, L is off; if K is off, L is on.

J and N cannot both be on.

If M is off, either J or N is on; if either J or N is on, M is off.

If P is on, L is on.

If O is off, N is off; if O is on, N is on.

Assume that the circuit is working as designed.

18. Any of the following can be true EXCEPT:

(A) J and O are both off.

(B) K and N are both off.

(C) K and P are both on.

(D) L and M are both on.

(E) N and O are both on.

19. If L and O are on, which one of the following must be true?

(A) J is on and K is off.

(B) J is off and N is on.

(C) K is off and M is on.

(D) K is off and P is on.

(E) M is off and P is on.

20. Which one of the following can be true?

(A) Only J, K, and M are off.

(B) Only J, L, and M are off.

(C) Only K, M, and O are off.

(D) Only L, N, and O are off.

(E) Only M, N, and O are off.

21. If P is on, what is the maximum number of lights that can be off?

(A) 2

(B) 3

(C) 4

(D) 5

(E) 6

22. What is the minimum number of lights that must be on?

(A) 0

(B) 1

(C) 2

(D) 3

(E) 4

23. If J is on, which one of the following could be a complete and accurate list of the remaining lights that are also on?

(A) K, L, and M

(B) L, N, and P

(C) L, O, and P

(D) M, N, and O

(E) L and P

Key : CDEBB EAEBD CDCED BACBA CCE

SECTION 2

Time-35 minutes

24 Questions

Directions: Each group of questions in this section is based on a set of conditions. In answering some of the questions, it may be useful to draw a rough diagram. Choosethe response that most accurately and completely answers each question and blacken the corresponding space on your answer sheet.

Question 1-6

Eight new students-R, S, T, V, W, X, Y, Z-are being divided among exactly three classes-class 1, class 2, and class3. Classes 1 and 2 will gain three new students each:

class 3 will gain two new students. The following restrictions apply:

R must be added to class 1.

S must be added to class 3.

Neither S nor W can be added to the same class as Y.

V cannot be added to the same class as Z.

If T is added to class 1, Z must also be added to class 1.

1.Which one of the following is an acceptable assignment of students to the three classes?

1 2 3

(A) R, T, YV, W, XS, Z

(B) R, T, ZS, V, YW, X

(C) R, W, XV, Y, XS, T

(D) R, X, ZT, V, YS, W

(E) R, X, ZV, W, YS, T

2.Which one of the following is a complete and accurate list of classes any one of which could be the class to which V is added?

(A) class 1

(B) class 3

(C) class 1, class 3

(D) class 2, class 3

(E) class 1, class 2, class 3.

3.If X is added to class 1, which one of the following is a student who must be added to class 2?

(A) T

(B) V

(C) W

(D) Y

(E) Z

4.If X is added to class 3, each of the following is a pair of students who can be added to class 1 EXCEPT

(A) Y and Z

(B) W and Z

(C) V and Y

(D) V and W

(E) T and z

5.If T is added to class 3, which one of the following is a student who must be added to class 2?

(A) V

(B) W

(C) X

(D) Y

(E) Z

6.Which one of the following must be true?

(A) If T and X are added to class 2. V is added to class 3.

(B) If V and W are added to class 1. V is added to class 3.

(C) If V and W are added to class 1. V is added to class 3.

(D) If V and X are added to class 1. W is added to class 3.

(E) If Y and Z are added to class 2. V is added to class 2.

Question 7-12

Four lions-F, G, H, J-and two tigers-K and M-will be assigned to exactly six stalls, one animal per stall. The stalls are arranged as follows:

First Row: 1 2 3

Second Row: 4 5 6

The only stalls that face each other are stalls 1 and 4, stalls 2 and 5, and stalls 3 and 6. The following conditions apply:

The tigers' stalls cannot face each other.

A lion must be assigned to stall 1

H must be assigned to stall 6.

J must be assigned to a stall numbered one higher than K's stall.

K cannot be assigned to the stall that faces H's stall.

7. Which one of the following must be true?

(A) F is assigned to an even-numbered stall

(B) F is assigned to stall 1

(C) J is assigned to stall 2 or else stall 3

(D) J is assigned to stall 3 or else stall 4

(E) K is assigned to stall 2 or else stall 4

8. Which one of the following could be true?

(A) F's stall is numbered one higher than J's stall

(B) H's stall faces M's stall

(C) J is assigned to stall 4

(D) K's stall faces J'S stall

(E) K's stall is in a different row than J's stall

9. Which one of the following must be true?

(A) A tiger is assigned to stall 2

(B) A tiger is assigned to stall 5

(C) K's stall is in a different row from M's stall

(D) Each tiger is assigned to an even-numbered stall

(E) Each lion is assigned to a stall that faces a tiger is stall

10. If K's stall is in the same row as H's stall which one of the following must be true?

(A) F's stall is in the same row as J's stall

(B) F is assigned to a lower-numbered stall than G

(C) G is assigned to a lower-numbered stall than M

(D) G's stall faces H's stall

(E) M's stall is in the same row as G's stall

11. If J is assigned to stall 3, which one of the following could be true?

(A) F is assigned to stall 2

(B) F is assigned to stall 4

(C) G is assigned to stall 1

(D) G is assigned to stall 4

(E) M is assigned to stall 5

12. Which one of the following must be true

(A) A tiger is assigned to stall 2

(B) A tiger is assigned to stall 4

(C) A tiger is assigned to stall 5

(D) A lion is assigned to stall 3

(E) A lion is assigned to stall 4

Question 13-18

On an undeveloped street, a developer will simultaneously build four houses on one side, numbered consecutively 1, 3, 5, and 7, and four houses on the opposite side,

numbered consecutively 2, 4, 6, and 8. Houses 2, 4, 6, and 8 will face houses 1, 3, 5, and 7, respectively. Each house will be exactly one of three styles-ranch, split-level or

Tudor-according to the following conditions:

Adjacent houses are of different styles.

No split-level house faces another split-level house.

Every ranch house has at least one Tudor house adjacent to it

House 3 is a ranch house.

House 6 is a split-level house.

13. Any of the following could be a Tudor house EXCEPT house

(A) 1

(B) 2

(C) 4

(D) 7

(E) 8

14. If there is one ranch house directly opposite another ranch house, which one of the following could be true?

(A) House 8 is a ranch house

(B) House 7 is a split-level house.

(C) House 4 is a Tudor house.

(D) House 2 is a split-level house.

(E) House 1 is a ranch house.

15. If house 4 is a Tudor house, then it could be true that house

(A) 1 is a Tudor house

(B) 2 is a Tudor house

(C) 5 is a Tudor house

(D) 7 is a Tudor house

(E) 8 is a Tudor house

16. On the street, there could be exactly.

(A) one ranch house

(B) one Tudor house

(C) two Tudor houses

(D) four ranch houses

(E) five ranch houses

17. If no house faces a house of the same style, then it must be true that

house

(A) 1 is a split-level house

(B) 1 is a Tudor house

(C) 2 is a ranch house

(D) 2 is a split-level house

(E) 4 is a Tudor house

18. If the condition requiring house 6 to be a split-level house is suspended but all other original conditions remain the same, the any of the following could be an accurate

list of the styles of houses 2, 4, 6, and 8, respectively, EXCEPT:

(A) ranch, split-level, ranch, Tudor

(B) split-level, ranch, Tudor, split-level

(C) split-level, Tudor, ranch, split-level

(D) Tudor, ranch. Tudor, split-level

(E) Tudor split-level, ranch, Tudor

Questions 19-24

Within a tennis league each of five teams occupies one of five positions, numbered 1 through 5 in order of rank, with number 1 as the highest position. The teams are

initially in the order R, J, S, M, L, with R in position 1. Teams change positions only when a lower-positioned team defeats a higher-positioned team. The rules are as

follows:

Matches are played alternately in odd-position rounds and in even-position rounds.

In an odd-position round, teams in position 3 and 5 play against teams positioned immediately above them.

In an even-position round, teams in position 2 and 4 play against teams positioned immediately above them.

When a lower-positioned team defeats a higher-positioned team, the two teams switch positions after the round is completed.

19. Which one of the of following could be the order of teams, from position 1 through position 5 respectively, after exactly one round of even-position matches if no oddposition

round has yet been played?

(A) J , R, M, L, S

(B) J, R, S, L, M

(C) R, J, M, L, S

(D) R, J, M, S, L

(E) R, S, J, L, M

20. If exactly two rounds of matches have been played, beginning with an odd-position round, and if the lower-positioned teams have won every match in those two rounds,

then each of the following must be true EXCEPT:

(A) L is one position higher than J.

(B) R is one position higher than L.

(C) S is one position higher than R.

(D) J is in position 4.

(E) M is position 3.

21. Which one of the following could be true after exactly two rounds of matches have been played?

(A) J has won two matches.

(B) L has lost two matches.

(C) R has won two matches.

(D) L's only match was played against J.

(E) M played against S in two matches.

22. If after exactly three rounds of matches M is in position 4, and J and L have won all of their matches, then which one of the following can be true?

(A) J is in position 2.

(B) J is in position 3.

(C) L is in position 2.

(D) R is in position 1.

(E) S is in position 3.

23. If after exactly three rounds M has won three matches and the rankings of the other four teams relative to each other remain the same, then which one of the following

must be in position 3?

(A) J

(B) L

(C) M

(D) R

(E) S

24. If after exactly three rounds the teams, in order from first to fifth position, are R, J, L, S, and M, then which one of the following could be the order, from first to fifth position,

of the teams after the second round?

(A) J, R, M, S, L

(B) J, L, S, M, R

(C) R, J, S, L, M

(D) R, L, M, S, J

(E) R, M, L, S, J

DEAEC DEBCE CBDBA AEADE ACAC

SECTION 3

Time-35 minutes

24 Questions

Questions 1-6

Seven students-fourth-year students Kim and Lee; third-year students Pat and Robin: and second-year students Sandy, Tety and Val-and only those seven, are

being assigned a rooms of equal size in a dormitory. Each room assigned must have either one or two or three students assigned to it and will accordingly be called either a

single or a double or a triple. The seven students are assigned to moms in accordence with the following conditions:

Lio fourth-year student can be assigned to a triple.

No second-year student can be assigned to a single.

Lee and Pobin must not share the same room

Kim and Pat must share the same room.

1. Which one of the following is a combination of rooms to which the seven students could be assigned?

(A) two triples and one single

(B) one triple and four singles

(C) three doubles and a stngle

(D) two doubles and three singles

(E) one double and five singles

2. It the room assigned to Robin is a single, which one of the following could be true?

(A) There is exactly one double that has a second-year student assigned to it.

(B) Lee is assigned to a stngle.

(C) Sandy Fat and one other student are zseigned to a triple together.

(D) Lixactly three of the rooms assigned to the students are singles

(E) Exactly two of the rooms assigned to the students are doubles.

3. Which one of the following must be true?

(A) Lee is assigned to a single

(B) Pat sharts a double with another student

(C) Robin shares a double with another student

(D) Two of the second-year students share a double with each other

(E) Neither of the third-year students is assigned to a single

4. If Robin is assigred to a triple, which one of the following must be true?

(A) Lee is assigned to a single

(B) Two second-year students share a double with each other

(C) None of the rooms assigned to the students is a single

(D) Two of the rooms assigned to the students are singles.

(E) Three of the rooms assigned to the students are singles

5. If Terry and Val assigned to different doubles from each other, other, then it must be true of the students' rooms that exactly

(A) one is a single

(B) two are singles

(C) two are doubles

(D) one is a triple